Nombre de nombres

Bonjour

si $n$ est entier, et $\dfrac{p}{q}$ un rationnel avec $p\wedge q=1$, quelqu'un peut-il, en fonction de $p$, $q$ et $n$, me dire combien a-t-on de nombres de la forme :

$\dfrac{p}{q} x - \left\lfloor \dfrac{p}{q}x \right\rfloor$ lorsque $x$ entier varie dans $[0,n]$,

Et si ce n'est pas possible de donner le nombre exact, alors de majorer très finement (mieux que $\min(q,m)$), avec une borne faisant intervenir $p$ ?

Merci

Réponses

  • Que reproches-tu à la majoration $\min(q,n)$ ? Peux-tu trouver un exemple où on fait mieux ?
    [size=x-small]Il me semble savoir montrer qu'on ne fait pas mieux.[/size]
  • Comme p intervient dans le problème, j'aimerais pouvoir le faire intervenir dans une borne plus précise

    Il y a des cas où cette borne sans $p$ n'est pas atteinte

    Je voudrais ainsi faire intervenir $p$ ou bien trouver plus fin
  • Il y a des cas où cette borne sans $p$ n'est pas atteinte

    Peux-tu m'en montrer ?
  • pour le moment non.

    Peut-on avoir une égalité alors ?

    Si non, cela ne suggère-t-il pas que l'on puisse faire mieux que $\min (q,n)$ ?
  • Alors, reviens râler contre la borne quand tu auras un exemple qu'on peut faire mieux ! :-D
    Et si tu n'en trouves pas, montre que le nombre de valeurs différentes de $\dfrac{px}{q} -\left\lfloor \dfrac{px}{q}\right\rfloor$ pour $x$ entier dans $[0,n]$ est égal à $\min(n,q)$.
  • ... a écrit:
    GaBuZoMeu écrivait:
    > Alors, reviens râler contre la borne quand tu
    > auras un exemple qu'on peut faire mieux ! :-D
    >

    Je ne t'ai pas obligé de m'aider.
    Change de ton.
  • Personne n'a raison contre un enfant qui pleure.


  • Pour $n,p,q\in\N$ avec $p\wedge q=1$ , on a posé \[
    \mu(n,p,q)=Card \left\{\left. \dfrac{p}{q} x - \left\lfloor \dfrac{p}{q}x \right\rfloor \,\,\, \right| \,\,\, x \in [0,n], x\in \N \right\}
    \]

    J'ai l'impression que $\mu(n,p,q)= \min (n+1, q) + 0 \times p$. N'est-ce pas une magnifique formule, contenant $p$ ?

    Cordialement, Pierre.
  • Oui, entre 0 et $n$ ça fait $n+1$.
    newser, je t'avais prévenu dès le début. Tu devrais être un peu plus cool.
  • ok merci
    J'aurais pensé à une formule avec p explicitement

    Une idée pour démontrer la formule de pldx1 sachant que l'on a déjà le sens $\mu (n,n,p)\leq \min (n+1, q)$ ?

    merci
  • Montrer que $ \dfrac{p}{q} x - \left\lfloor \dfrac{p}{q}x \right\rfloor= \dfrac{p}{q} y - \left\lfloor \dfrac{p}{q}y \right\rfloor$ (avec $x$ et $y$ entiers) si et seulement si $q$ divise $x-y$ ?
    La division euclidienne de $px$ par $q$ peut servir.
  • Merci GaBuZoMeu
  • @GaBuZomeu, ton argument est-il suffisant ?
Connectez-vous ou Inscrivez-vous pour répondre.